Write an equation that expresses the following relationship.
w varies directly with u and inversely with the square of d
In your equation, use k as the constant of proportionality.

Answers

Answer 1

Answer:

w = [tex]\frac{ku}{d^{2} }[/tex]

Step-by-step explanation:

Answer 2

An equation that expresses the given relationship is ud²=1.

Given that, w varies directly with u and inversely with the square of d.

We need to write an equation that expresses the following relationship.

What is directly and inversely varies?

Direct variation is a linear function defined by an equation of the form y = kx when x is not equal to zero. Inverse variation is a nonlinear function defined by an equation of the form xy = k when x is not equal to zero and k is a nonzero real number constant.

Now, w∝u⇒w=ku

and w∝1/d²⇒w=k/d²

⇒wd²=k

⇒w=wd²u

ud²=1

Therefore, an equation that expresses the given relationship is ud²=1.

To learn more about varies directly and inversely visit:

https://brainly.com/question/2315806.

#SPJ2


Related Questions

If the areas of the given pairs of shapes are equal, find the value of x.​

Answers

Answer:

X= 8cm

Step-by-step explanation:

area of the square = s×s = 16×16 = 256cm

area of the rectangle = l×b = 32×x = 32x

Given , area of triangle = area of square

32x = 256

x= 256/32

x= 8cm

Answer:

[tex]x = 8cm[/tex]

Step-by-step explanation:

we are given a square and rectangle.we want to figure out x which is the width of the rectangle. we are also given a condition i.e

the area of the square equal to the area of the rectangle

therefore,

[tex] \displaystyle \rm A _{square } = A _{rect}[/tex]

recall the formula of the area of square and rectangle so,

[tex] \displaystyle {s}^{2} = lw[/tex]

now assign variables

[tex]s \implies16cm[/tex][tex]l \implies32cm[/tex][tex]w \implies x[/tex]

thus substitute:

[tex] \displaystyle 32cmx = {16cm}^{2} [/tex]

simplify square:

[tex] \displaystyle 32cmx = 256cm^2[/tex]

divide both sides by 32:

[tex] \displaystyle \boxed{x = 8cm}[/tex]

and we're done!

Which option is correct and how would one solve for it?

Answers

Answer:

102

Step-by-step explanation:

We have the sum for k = 1 to 4 of 3 ^ ( k-1)  * ( k-1)

k =1   3 ^ (1-1) * ( 1-1) = 3^0 * 0 = 0

k =2   3 ^ (2-1) * ( 2-1) = 3^1 * 1 = 3

k =3   3 ^ (3-1) * ( 3-1) = 3^2 * 2 = 9*2 = 18

k =4   3 ^ (4-1) * ( 4-1) = 3^3 * 3 = 27 *3 = 81

Add these together

0+3+18+81 =102

━━━━━━━☆☆━━━━━━━

▹ Answer

102

▹ Step-by-Step Explanation

Convert the notation into a sum and substitute values from 1-4:

(3¹⁻¹ *(1 - 1)) + (3²⁻¹ * (2 - 1)) + (3³⁻¹ * (3 - 1)) + (3⁴⁻¹ * (4 - 1))

0 + 3 + 18 + 81

= 102

Hope this helps!

CloutAnswers ❁

━━━━━━━☆☆━━━━━━━

if sin150=1/2 then find sin75

Answers

Answer:

0.966

Step-by-step explanation:

When typed into a calculator, sin75 = -0.3877816354

Upon converting to degrees, the full answer is 0.96592582628

Write an equation showing the relationship between the lengths of the three sides of a right triangle.​

Answers

Answer:

Below

Step-by-step explanation:

First triangle)

This triangle is a right one so we will apply the pythagorian theorem.

● 25 is the hypotenus

● 25^2 = b^2 + 24^2

■■■■■■■■■■■■■■■■■■■■■■■■■■

Seconde triangle)

Again it's a right triangle

x is the hypotenus.

● x^2 = 12^2 +5^2

● 12^2 = x^2-5^2

■■■■■■■■■■■■■■■■■■■■■■■■■■

This is a right triangle

AC is the hypotenus.

● AC^2 = BC^2 + BA^2

Notice that: BC = BE+EC and BA=BD+DA

● AC^2 = (BE+EC)^2 + (BD+DA)^2

Answer:  2) b = 7       3) x = [tex]\sqrt{119}[/tex]

Step-by-step explanation:

Use Pythagorean Theorem: (leg₁)² + (leg₂)² = hypotenuse²

2) b² + 24² = 25²

   b² + 576 = 625

            b² = 49

         [tex]\sqrt{b^2}=\sqrt{49}[/tex]

            b = 7

3) 5² + x² = 12²

   25 + x² = 144

            x² = 119

        [tex]\sqrt{x^2}=\sqrt{119}[/tex]

           [tex]x=\sqrt{119}[/tex]

a ball is thrown upward with an initial height of 3 feet with an initial upward velocity 37 ft/s the balls heigh in feet after t second is given by h=3=+37t-16t^2

Answers

Answer:

[tex]t = 1.45[/tex] or [tex]t = 0.86[/tex]

Step-by-step explanation:

Given

[tex]h=3+37t-16t^2[/tex]

Required

Find all values of t when height is 23 feet

To solve this, we simply substitute 23 for h

[tex]23=3+37t-16t^2[/tex]

Collect  like terms

[tex]16t^2 - 37t - 3 + 23=0[/tex]

[tex]16t^2 - 37t +20=0[/tex]

Solve t using quadratic formula;

[tex]t = \frac{-b\±\sqrt{b^2 - 4ac}}{2a}[/tex]

Where a = 16, b =-37 and c = 20

[tex]t = \frac{-(-37)\±\sqrt{(-37)^2 - 4*16*20}}{2*16}[/tex]

[tex]t = \frac{37\±\sqrt{(-37)^2 - 4*16*20}}{2*16}[/tex]

[tex]t = \frac{37\±\sqrt{1369 - 1280}}{32}[/tex]

[tex]t = \frac{37\±\sqrt{89}}{32}[/tex]

[tex]t = \frac{37\±9.43}{32}[/tex]

[tex]t = \frac{37+9.43}{32}[/tex] or [tex]t = \frac{37-9.43}{32}[/tex]

[tex]t = \frac{46.43}{32}[/tex] or [tex]t = \frac{27.57}{32}[/tex]

[tex]t = \frac{46.43}{32}[/tex] or [tex]t = \frac{27.57}{32}[/tex]

[tex]t = 1.45[/tex] or [tex]t = 0.86[/tex]

consider the bevariate data below about Advanced Mathematics and English results for a 2015 examination scored by 14 students in a particular school.The raw score of the examination was out of 100 marks.
Questions:
a)Draw a scatter graph
b)Draw a line of Best Fit
c)Predict the Advance Mathematics mark of a student who scores 30 of of 100 in English.
d)calculate the correlation using the Pearson's Correlation Coefficient Formula
e) Determine the strength of the correlation

Answers

Answer:

Explained below.

Step-by-step explanation:

Enter the data in an Excel sheet.

(a)

Go to Insert → Chart → Scatter.

Select the first type of Scatter chart.

The scatter plot is attached below.

(b)

The scatter plot with the line of best fit is attached below.

The line of best fit is:

[tex]y=-0.8046x+103.56[/tex]

(c)

Compute the value of x for y = 30 as follows:

[tex]y=-0.8046x+103.56[/tex]

[tex]30=-0.8046x+103.56\\\\0.8046x=103.56-30\\\\x=\frac{73.56}{0.8046}\\\\x\approx 91.42[/tex]

Thus, the Advance Mathematics mark of a student who scores 30 out of 100 in English is 91.42.

(d)

The Pearson's Correlation Coefficient is:

[tex]r=\frac{n\cdot \sum XY-\sum X\cdot \sum Y}{\sqrt{[n\cdot \sum X^{2}-(\sum X)^{2}][n\cdot \sum Y^{2}-(\sum Y)^{2}]}}[/tex]

  [tex]=\frac{14\cdot 44010-835\cdot 778}{\sqrt{[14\cdot52775-(825)^{2}][14\cdot 47094-(778)^{2}]}}\\\\= -0.7062\\\\\approx -0.71[/tex]

Thus, the Pearson's Correlation Coefficient is -0.71.

(e)

A correlation coefficient between ± 0.50 and ±1.00 is considered as a strong correlation.

The correlation between Advanced Mathematics and English results is -0.71.

This implies that there is a strong negative correlation.

the temp fell 3 degrees every hour for 5 hours what's the change in temperature​

Answers

Answer:

-15

Step-by-step explanation:

If it fell 3 deg every hour for 5 hours so the equation is 3*5 plus a - sign because it dropped degrees

help please i need this now step by step

Answers

Answer:

40 mi ^2

Step-by-step explanation:

SA = 2(lw+wh+lh)  where l is the length, w is the width, h is the height

SA = 2( 2*2 + 2*4 * 2*4)

    = 2( 4+8+8)

    = 2(20)

    = 40 mi ^2

NEED IT ASAP
Tony is shopping for new tires for his 4-wheel-drive truck. In addition to the price of the tires, there is a 10% sales tax plus a state-mandated $45 fee for disposing of his old tires. If Tony has determined that he will spend less than $559.80 total, then what is the price range he can spend on the tire set?
Select one:
a. Less than $468
b. At least 472
c. $468 or more
d. Less than 473

Answers

Answer:

A

Step-by-step explanation:

Let:

Total price be T

And price of tire set be x

T<559.80 ---(1)

T=x +(10% of x)+ 45. ——(2)

T=x+(1/10)x+45

T=(11/10)x+45

Substitute T into equ. 1

T<559.80

(11/10)x +45<559.80

(11/10)x < 514.80

11x < 5148

x < 468

Examining and Interpreting a Statistical Report
To help make his case to potential investors in his invention, Martin found a poll conducted by Gallup, a well-known research organization. Take some time to read the report, and then answer the questions to analyze the merits and limitations of this survey.

Part A
The poll report includes a table titled, “Americans Using Cash Now Versus Five Years Ago, by Age.” The age intervals are not equal. Why do you think the Gallup organization chose the age intervals of 23–34, 35–54, and 55+ to display these results?

Answers

The Gallup organization chose the age intervals of 23–34, 35–54, and 55+ to display these results due to they wanted to identify the habits about the use of cash in the different stages of a person's life, differentiated by ranges of unequal age.

Application of statistics in the collection of information.

Although the most common case when preparing a frequency table is to develop equal ranges that allow the behavior of the respondents to be identified every certain number of possible values.

Example:

0.1 - 10 10.1 - 20 20.1 - 30

On some occasions, irregular or uneven ranges are used that refer to a specific market stage or niche.

In part A of the exercise, you can clearly identify the three stages of adult life that could change your money-use habits:

23 - 34: Group that could be called "young adults", possibly made up of university students, and people who recently started their work stage. 35 - 54: Group of "middle-aged people", made up of people who have been working and managing their finances for a long time, even those who are even close to retirement. 55 or more: People of the "third age" or close to said category, made up of pre-pensioners and pensioners or people who are no longer active in employment.

As you can see, each of these groups present great differences that, from the point of view of a person interested in investing in their invention, a poll report like this is more attractive to select the niche of potential investors based on the work stage of said person.

By this reason, The Gallup organization chose the age intervals due to their money-habits, depending on the stage of his/her life.

If you want to see another example about poll reports, you can use the next link: https://brainly.com/question/15223266?referrer=searchResults

5/2 + 6g = 11/4 solve it

Answers

Answer:

g = [tex]\frac{1}{24}[/tex]

Step-by-step explanation:

Given

[tex]\frac{5}{2}[/tex] + 6g = [tex]\frac{11}{4}[/tex]

Multiply through by 4 to clear the fractions

10 + 24g = 11 ( subtract 10 from both sides )

24g = 1 ( divide both sides by 24 )

g = [tex]\frac{1}{24}[/tex]

WILL GIVE BRAINILY 5 STARS AND THANKS FOR CORRECT ANSWERLenora wants to buy granola bars for her hiking trip. Eight bars cost $2.40. She wants to buy 12 bars. How much will 12 bars cost?

Answers

Answer:

$3.60

Step-by-step explanation:

2.40/8=.30

$.30 per bar

.3 x 12= $3.60

Answer:

$3.60

Step-by-step explanation:

Proportions

8 bars ⇔ $2.4

12 bars ⇔ $W

W = 12*2.4/8

W = $3.6

Find the limit. Use l'Hospital's Rule where appropriate. If there is a more elementary method, consider using it. lim x→9 x − 9 x2 − 81

Answers

Without resorting to L'Hopitâl's rule,

[tex]\displaystyle\lim_{x\to9}\frac{x-9}{x^2-81}=\lim_{x\to9}\frac{x-9}{(x-9)(x+9)}=\lim_{x\to9}\frac1{x+9}=\frac1{18}[/tex]

With the rule, we get the same result:

[tex]\displaystyle\lim_{x\to9}\frac{x-9}{x^2-81}=\lim_{x\to9}\frac1{2x}=\frac1{18}[/tex]

Please help. I’ll mark you as brainliest if correct

Answers

Answer:

(a)

dependent

(b)

x = -3t - 12

y = -5t - 16

z = t

Step-by-step explanation:

2x - 3y - 9z = 24       Eq. 1

x + 3z = -12                Eq. 2

-3x + y - 4z = 20        Eq. 3

      2x - 3y - 9z = 24

(+)  -9x + 3y - 12x = 60     3 * Eq. 3

--------------------------------

     -7x           -21z = 84     Eq. 4

       7x + 21z = -84          7 * Eq. 2

(+)    -7x - 21z = 84            Eq. 4

-----------------------------

                  0  = 0

(a) The system is dependent.

(b)

z = t

x + 3z = -12                Eq. 2

x + 3t = -12

x = -3t - 12

2x - 3y - 9z = 24       Eq. 1

2(-3t - 12) - 3y - 9t = 24

-6t - 24 - 3y - 9t = 24

-3y - 15t = 48

-y - 5t = 16

-y = 5t + 16

y = -5t - 16

x = -3t - 12

y = -5t - 16

z = t

A rotating light is located 16 feet from a wall. The light completes one rotation every 2 seconds. Find the rate at which the light projected onto the wall is moving along the wall when the light's angle is 20 degrees from perpendicular to the wall.

Answers

Answer:

a

Step-by-step explanation:

answer is a on edg

$17,818 is invested, part at 11% and the rest at 6%. If the interest earned from the amount invested at 11% exceeds the interest earned from the amount invested at 6% by $490.33, how much is invested at each rate? (Round to two decimal places if necessary.)

Answers

Answer:We know the total amount of money invested. $17818

x+y=17818,

We know that the difference in interest earned by the two accounts is $490.33

0.11*x-0.06*y=490.33

x=17818-y

We substitute for x

0.11*(17818-y)-0.06*y=490.33

We multiply out

1959.98-0.11y-0.06*y=490.33

We combine like terms.

1469.65=0.17*y

Isolate y

y=1469.65/0.17

y=8645 at 6%

Calculate x

x=17818-8645

x=9173 at 11%

Check

0.11*9173-0.06*8645=490.33

interest earned at 11%=1009.03

interest earned at 6%=518.70

1009.03-518.7=490.33

490.33=490.33

Since this statement is TRUE and neither amount is negative then all is well.We know the total amount of money invested. $17818

x+y=17818,

We know that the difference in interest earned by the two accounts is $490.33

0.11*x-0.06*y=490.33

x=17818-y

We substitute for x

0.11*(17818-y)-0.06*y=490.33

We multiply out

1959.98-0.11y-0.06*y=490.33

We combine like terms.

1469.65=0.17*y

Isolate y

y=1469.65/0.17

y=8645 at 6%

Calculate x

x=17818-8645

x=9173 at 11%

Check

0.11*9173-0.06*8645=490.33

interest earned at 11%=1009.03

interest earned at 6%=518.70

1009.03-518.7=490.33

490.33=490.33

Since this statement is TRUE and neither amount is negative then all is well.

John painted his most famous​ work, in his​ country, in 1930 on composition board with perimeter 101.14 in. If the rectangular painting is 5.43 in. taller than it is​ wide, find the dimensions of the painting.

Answers

Answer:

22.57 x 28

Step-by-step explanation:

10.86 + 4x = 101.14

-10.86           -10.86

            4x = 90.28

             /4       /4

              x = 22.57

5.43 + 22.57 = 28

22.57

Line A passes through the point (-1,2). Which of the
following CANNOT be the equation of line A?
A y=1 - 2
B
y = x +1
C
X = -1
D y=x+3

Answers

Answer:

b

Step-by-step explanation:

y = x + 1

The correct answer is (B). The slope-intercept form of a line is y = mx + b. Since the line passes through (−1,2), there are three possibilities: the line will have a slope (the "m" in front of the "x" variable), it will be vertical (x = −1), or it will be horizontal (y = 2). Plug x = −1 into all four equations to see which equation is not satisfied. The only answer choice that doesn't give us y = 2 is (B).

Option B is correct.

Given:

Line A passes through the point [tex](-1,2)[/tex].

To find:

Which of the given equations cannot be the equation of line A.

Solution:

If Line A passes through the point [tex](-1,2)[/tex], it means the equation of Line A must be satisfied by the point

In option A, consider the given equation is:

[tex]y=1-x[/tex]

Substituting [tex]x=-1,y=2[/tex], we get

[tex]2=1-(-1)[/tex]

[tex]2=1+1[/tex]

[tex]2=2[/tex]

This statement is true. So, [tex]y=1-x[/tex] can be the equation of line A.

Similarly, check for the other options.

In option B,

[tex]y=x+1[/tex]

Substituting [tex]x=-1,y=2[/tex], we get

[tex]2=-1+1[/tex]

[tex]2=0[/tex]

This statement is false. So, [tex]y=x+1[/tex] cannot be the equation of line A.

In option C,

[tex]x=-1[/tex]

It is a vertical line and it passes through the point [tex](-1,2)[/tex] because the x-coordinate is [tex]-1[/tex]. So, [tex]x=-1[/tex] can be the equation of line A.

In option D,

[tex]y=x+3[/tex]

Substituting [tex]x=-1,y=2[/tex], we get

[tex]2=-1+3[/tex]

[tex]2=2[/tex]

This statement is true. So, [tex]y=x+3[/tex] can be the equation of line A.

Therefore, the correct option is B.

Learn more:

https://brainly.com/question/13078415

A box is dragged across 20 meters with a force of 60 Newtons, which are kg*m/s^2

Answers

Answer:

Mass= 6kg

Acceleration= 10 ms^-2

Work done = 1200Nm

Step-by-step explanation:

kg*m/s^2 represent the force.

The kg represent the mass

The m/s^2 represent the acceleration

The acceleration here will be due to gravity force= 10 ms^-2

Then the mass= 60/10

Mass= 6 kg

The force = 60 Newton

Distance covered in the direction of the the force= 20 Meters

The work done in the direction of the force= force* distance

The work done in the direction of the force=60*20

The work done in the direction of the force=1200 Nm

Answer: 20 • 60

Step-by-step explanation:

Find out the Time Zone for UAE and its neighboring countries. Express them as positive or negative rational numbers with reference to Greenwich Mean Time. Note down the time of few of your daily activities such as breakfast, school time, lunch time, etc. Compare the same time with GMT.anyone please answer this.

Answers

Answer:

UAE is in the Gulf Standard Time zone.

It is GMT + 4

Breakfast: 7 am; GMT 3 am

School time 8 am: GMT 4 am

Lunch time: 12:30 pm; GMT 8:30 am

Step-by-step explanation:

UAE is in the Gulf Standard Time zone.

It is GMT + 4

Breakfast: 7 am; GMT 3 am

School time 8 am: GMT 4 am

Lunch time: 12:30 pm; GMT 8:30 am

Given the trinomial, what is the value of the coefficient B in the factored form?
2x2 + 4xy − 48y2 = 2(x + By)(x − 4y)

Answers

Answer:

B = 6

Step-by-step explanation:

2x^2 + 4xy − 48y^2

Factor out 2

2(x^2 + 2xy − 24y^2)

What 2 numbers multiply to -24 and add to 2

-4 *6 = -24

-4+6 = 2

2 ( x+6y)( x-4y)

Answer:

[tex]\huge\boxed{B=6}[/tex]

Step-by-step explanation:

They are two way to solution.

METHOD 1:

Factor the polynomial on the left side of the equation:

[tex]2x^2+4xy-48y^2=2(x^2+2xy-24y^2)=2(x^2+6xy-4xy-24y^2)\\\\=2\bigg(x(x+6y)-4y(x+6y)\bigg)=2(x+6y)(x-4y)[/tex]

Therefore:

[tex]2x^2+4xy-48y^2=2(x+By)(x-4y)\\\Downarrow\\2(x+6y)(x-4y)=2(x+By)(x-4y)\to\boxed{\bold{B=6}}[/tex]

METHOD 2:

Multiply everything on the right side of the equation using the distributive property and FOIL:

[tex]2(x+By)(x-4y)=\bigg((2)(x)+(2)(By)\bigg)(x-4y)\\\\=(2x+2By)(x-4y)=(2x)(x)+(2x)(-4y)+(2By)(x)+(2By)(-4y)\\\\=2x^2-8xy+2Bxy-8By^2=2x^2+(2B-8)xy-8By^2[/tex]

Compare polynomials:

[tex]2x^2+4xy-48y^2=2x^2+(2B-8)xy-8By^2[/tex]

From here we have two equations:

[tex]2B-8=4\ \text{and}\ -8B=-48[/tex]

[tex]1)\\2B-8=4[/tex]        add 8 to both sides

[tex]2B=12[/tex]         divide both sides by 2

[tex]B=6[/tex]

[tex]2)\\-8B=-48[/tex]          divide both sides by (-8)

[tex]B=6[/tex]

The results are the same. Therefore B = 6.

Let f(x) and g(x) be polynomials as shown below. Which of the following is true about f(x) and g(x)? f(x) and g(x) are not closed under multiplication because when multiplied, the result will not be a polynomial. f(x) and g(x) are not closed under multiplication because when multiplied, the result will be a polynomial. f(x) and g(x) are closed under multiplication because when multiplied, the result will not be a polynomial. f(x) and g(x) are closed under multiplication because when multiplied, the result will be a polynomial. Reset Submit

Answers

Answer:

  f(x) and g(x) are closed under multiplication because when multiplied, the result will be a polynomial.

Step-by-step explanation:

The set of all polynomials is closed under addition, subtraction, and multiplication, because performing any of these operations on a pair of polynomials will give a polynomial result.

__

Comment on the question

The wording is a bit strange, because f(x) and g(x) are elements of a set (of polynomials), so cannot be said to be "closed." "Closed" is a property of a set with respect to some function, it is not a property of an element of the set.

Answer:

f(x) and g(x) are closed under multiplication because when multiplied, the result will be a polynomial.

Step-by-step explanation:

Its correct trust

henry incorrectly said the rate 1/5 pound/ 1/20 quart can be written as the unit rate 1/100 pound per quart. What is the correct unit rate? What error did Henry likely make?

Answers

Answer:

4 pounds per quart

Step-by-step explanation:

Henry divided by 20 instead of multiplying by 20.

1/5 pound is the numerator and 1/20 quart is the denominator. To make the denominator equal to 1 quart, you need to multiply by 20.

So 1/5 x 20 = 4 pounds.

Literally, a unit rate means a rate for one.

The unit rate is 4 pounds per quartHenry used the wrong arithmetic operator

The rate is given as:

[tex]\mathbf{Rate = \frac{1}{5}\ pound\ per\ \frac{1}{20}\ quart}[/tex]

Per means divide.

So, the expression becomes

[tex]\mathbf{Rate = \frac{1}{5}\ pound\ \div \frac{1}{20}\ quart}[/tex]

Express as products

[tex]\mathbf{Rate = \frac{1}{5}\ pound\ \times \frac{20}{1\ quart}}[/tex]

Simplify

[tex]\mathbf{Rate = \frac{1}{1}\ pound\ \times \frac{4}{1\ quart}}[/tex]

Rewrite as:

[tex]\mathbf{Rate = \frac{4\ pound}{1\ quart}}[/tex]

So, the unit rate is 4 pounds per quart

Henry's error is that: He multiplied 1/5 by 1/20, instead of dividing 1/5 by 1/20

Read more about unit rates at:

https://brainly.com/question/18065083

Which graph shows the polar coordinates (-3,-) plotted

Answers

Graph 1 would be the answer

The length of a rectangle is twice its width. If the area of the rectangle is 72in², find its perimeter

Answers

Let breadth be x

Length=2x

[tex]\\ \sf\longmapsto Area=Length\times Breadth[/tex]

[tex]\\ \sf\longmapsto 72=2x(x)[/tex]

[tex]\\ \sf\longmapsto 2x^2=72[/tex]

[tex]\\ \sf\longmapsto x^2=\dfrac{72}{2}[/tex]

[tex]\\ \sf\longmapsto x^2=36[/tex]

[tex]\\ \sf\longmapsto x=\sqrt{36}[/tex]

[tex]\\ \sf\longmapsto x=6[/tex]

Length=6×2=12inBreadth=6in

[tex]\\ \sf\longmapsto Perimeter=2(L+B)[/tex]

[tex]\\ \sf\longmapsto Perimeter=2(12+6)[/tex]

[tex]\\ \sf\longmapsto Perimeter=2(18)[/tex]

[tex]\\ \sf\longmapsto Perimeter=36in[/tex]

The midterm exam will have a total of 108 marks and there are twice as many 05 marks questions than 02 marks questions. Each question is worth either 02 marks or 05 marks”. Find out how many questions of each value there are in the online test?​

Answers

Answer: 9 (2 mark question) and 18 (5 marks questions)

Step-by-step explanation:

Given: Total mark = 108

Let the 2 marks questions be x

Therefore 5 mark question = 2x

ATQ

⇒5(2x)+2(x) = 108

⇒10x+2x =108

⇒12x = 108

⇒x = 108/12

⇒x = 9

Total two mark questions are 9 and 5 mark questions are 18.

please click thanks and mark brainliest if you like :)

7 1/4 x−x=9 3/8 HELLLLPPPPP PLLSSSS

Answers

-1.5

Step-by-step explanation:

So, you do 7.25 - 1 (because it is) and you get 6.25. Make it a fraction inton 25/4 and divide bu 75/8 (9 3/8 simplified) and you get -1.5 voila.

Answer:

x = 3/2

Step-by-step explanation:

7 1/4 = 7 + 1/4 = 28/4 + 1/4 = 29/4

9 3/8 = 9 + 3/8 = 72/8 + 3/8 = 75/8

then:

7 1/4 x = 29x/4

29x/4 - x = 75/8

29x/4 - 4x/4 = 75/8

25x/4 = 75/8

x = (75/8)/(25/4)

x = (75*4)/(8*25)

x = 300/200

x = 3/2

Checking:

(29/4)(3/2) = (29*3)(4*2) =  87/8

87/8 - 3/2 = 75/8

3/2 = 12/8

then:

87/( - 12/8 = 75/8

y''+y'=3 hihihihihhihihihihihihih

Answers

Answer:

Maaf Saya Tidak Tahu

Step-by-step explanation:

Maaf Maaf

Simplify the product. 7(–9)

Answers

Hello!

Any time there is a number directly outside another in parentheses, that means multiply!

So, you must multiply 7 and -9. Remember a positive times a negative is always a negative.

7*-9= -63

Answer:-63

Hope this helped! Comment if you have questions :)

Answer:

have a great day

god is great

a is inversely proportional to (b - 4).
If a = 8 and b = 22, express a in terms of b.

Answers

Answer:

Step-by-step explanation:

a is expressed in terms of b as a = 144/(b - 4).

If A is inversely proportional to (b - 4), we can express this relationship using the formula:

A = k/(b - 4),

where k is the constant of proportionality.

To determine the value of k, we can use the given information when a = 8 and b = 22:

8 = k/(22 - 4).

Simplifying the equation:

8 = k/18.

To isolate k, we multiply both sides of the equation by 18:

8 * 18 = k.

k = 144.

Now that we know the value of k, we can rewrite the equation in terms of b:

A = 144/(b - 4).

Therefore, a is expressed in terms of b as a = 144/(b - 4).

Learn more about proportional here

https://brainly.com/question/32890782

#SPJ2

Other Questions
Christopher has breakfast at a cafe and the cost of his meal is\$36.00$36.00dollar sign, 36, point, 00. Because of the service, he wants to leave a10\%10%10, percenttip.What is his total bill including tip? Only ------ percent of the food eaten is turned into its own body. Hickam Company makes one product, for which it has developed the following standard for labor: each unit should require 1.50 hours at $12/hour. In April, Hickam made 10,000 units, using 1.65 hours per unit at a cost of $11.50 per hour. What is the labor usage variance No, the graph suggests that the increase in adoptions from 2000 to 2005 was less significant than it actually is. No, the graph suggests that the increase in adoptions from 2000 to 2005 was more significant than it actually is. Yes, the graph fairly and accurately depicts the data in an objective manner. Clara travels from her home to Stoke. The distance from her home to Stoke is 100 miles. She travels at an average speed of 50 miles per hour. She stops for 20 minutes on the journey. Clara arrives in Stoke at 10:10 am. At what time did she leave home? Below are a list of costs and discounts for groceries. Round to the nearest dollar to estimate the total cost+ $12.34+ $5.07- $0.73+ $2.84- $1.50 what kind of sentence is this?If the members of our team had cooperated the opposition which was well trained but uninspiredcomplex sentencecompound sentencerun-on sentencesentence fragmentsimple sentence Savonarola was a reformer who was extremely puritanical. True False Bree Wakefield is a 19 year old college student, living in a dormitory on the campus of her university. She started to feel poorly and began to miss classes. She was diagnosed with a disease of the CNS ? A 2.0 m 4.0 m flat carpet acquires a uniformly distributed charge of 10 C after you and your friends walk across it several times. A 5.0 g dust particle is suspended in midair just above the center of the carpet.Required:What is the charge on the dust particle? what is the connotation of the word strange? help help helphelphelp help When Isaiah Company has fixed costs of $137,750 and the contribution margin is $29, the break-even point is wholesale market definition (a) Determine the capacitance of a Teflon-filled parallel-plate capacitor having a plate area of 1.80 cm2 and a plate separation of 0.010 0 mm. pF (b) Determine the maximum potential difference that can be applied to a Teflon-filled parallel-plate capacitor having a plate area of 1.80 cm2 and a plate separation of 0.010 0 mm. kV Consider this polynomial, where a is an unknown real number.p(t) = x^4 +5x^3 + ax^2 - 3x + 11The remainder of the quotient of P(x), and (x+ 1) is 17.Braulio uses synthetic division to find the value of a, and Zahra uses the remainder theorem to find the value of a. write a story to illustrate the saying prevention is better than cure What are the semicolons doing in this sentence One thousandth of the base unit in length :One millionth of the base unit in mass :One thousandth of the base unite in time : It keeps saying my answer is wrong after i identified the GCF as 3 but maybe I typed it wrong.